Đến nội dung

Oral1020 nội dung

Có 1000 mục bởi Oral1020 (Tìm giới hạn từ 28-04-2020)



Sắp theo                Sắp xếp  

#496135 Cho các số thực $x,y,z \in [1,2]$. Tìm GTLN của $3(x+y+z)...

Đã gửi bởi Oral1020 on 30-04-2014 - 18:26 trong Bất đẳng thức và cực trị

Xét $f(x)=x(3-y-z)+3(y+z)-yz$ với $x \in [1;2]$

Xét $f(1)=3+2y+2z-yz=(2-y)(z-2)+7 \le 7$

      $f(2)=6+x+y-yz=(y-1)(z-1)+7 \le 7$

Mà $f(x) \le max {f(1);f(2)} \le 7$

Vậy max $f(x)=7$ khi $x=2;y=2;z=1$ và $x=1;y=1;z=2$ và các hoán vị




#496110 cho x,y,z >0 và $x^{2}+y^{2}+z^{2}=201...

Đã gửi bởi Oral1020 on 30-04-2014 - 16:52 trong Bất đẳng thức và cực trị

bài 1:

Bạn đi chứng minh :
$P \ge \sqrt{3(x^2+y^2+z^2)}$ (1)

Bằng cách bình phương hai vế và rút gọn ta còn lại bất đẳng thức sau:

$\dfrac{x^2y^2}{z^2}+\dfrac{y^2z^2}{x^2}+\dfrac{x^2z^2}{y^2} \ge x^2+y^2+z^2$

$\Longrightarrow \sum \dfrac{ab}{c} \ge \sum a$

Với $a=x^2;...$

Bất đẳng thức trên chứng minh được bằng cách

$BĐT \longleftrightarrow abc.\sum \dfrac{1}{a^2} \ge abc.\sum \dfrac{1}{ab}=VT$

=> (1) đúng.Áp dụng giả thiết ta có đpcm




#496031 Chứng minh $(ap+bc)(bp+ac)(cp+ab)$ là số chính phương

Đã gửi bởi Oral1020 on 30-04-2014 - 08:20 trong Số học

Ta có:

$a(a+b+c)+bc=a^2+ab+bc+ac=(a+b)(a+c)$

Tương tự,ta được:

$A= (a+b)^2(b+c)^2(c+a)^2$ là một số chính phương




#495878 Giải phương trình: $4\sqrt{x-1}=32x^{4}-80x^...

Đã gửi bởi Oral1020 on 29-04-2014 - 14:35 trong Số học

Giải phương trình: $4\sqrt{x-1}=32x^{4}-80x^{3}+50x^{2}+4x-3$.

(Mời các bạn THCS thảo luận)

Bạn xem tại đây




#495795 $\left\{\begin{matrix} x^{3}+4y=y^{3}+16x\\y^{2...

Đã gửi bởi Oral1020 on 28-04-2014 - 22:05 trong Phương trình - hệ phương trình - bất phương trình

Xét $(x;y)=(0;0)$ không thỏa mãn.

Xét $(x;y)$ khác (0;0),ta có:

$y^2=5x^2+4$

$\Longrightarrow y^3=5x^2y+4y$

Thay vào phương trình đầu,ta được:

$x^3+4y=5x^2y+4y+16x$

$\Rightarrow x^2=5xy+16$ hoặc x=0 => ...

Kết hợp với pt thứ 2 ta được hệ đẳng cấp




#495535 $\sqrt{x+5}=x^2+5$

Đã gửi bởi Oral1020 on 27-04-2014 - 20:10 trong Phương trình - hệ phương trình - bất phương trình

Bình phương và thu gọn,ta được:

$x^4+10x^2-x+20=0$

Áp dụng bất đẳng thức AM-GM,ta có
$VT \ge 12x^2-x+19 > 0$

Do vậy,pt đã cho vô nghiệm




#495316 Chứng minh rằng :$\sum \dfrac{a^2}{b}...

Đã gửi bởi Oral1020 on 26-04-2014 - 21:07 trong Bất đẳng thức và cực trị

Cho $a;b;c >0$ thỏa mãn $a+b+c=1$.Chứng minh rằng:
$\dfrac{a^2}{b}+\dfrac{b^2}{c}+\dfrac{c^2}{a} \ge 3a^2+3b^2+3c^2$

P/s:Càng nhiều cách càng tốt nhé ! :D




#495284 Tìm đa thức bậc hai sao cho f(x)-f(x-1)=x

Đã gửi bởi Oral1020 on 26-04-2014 - 18:57 trong Đại số

Giả sử $f(x)=ax^2+bx+c$ (do đề bài cho là đa thức bậc hai)
Suy ra

$f(x)-f(x-1)=ax^2+bx+c-a(x-1)^2-b(x-1)-c=2ax+a+b$

Mà $f(x)-f(x-1)=x$

$\Rightarrow 2ax+a+b=x$

Do đó $a+b =0$ và $a=1/2$ từ đó ta suy ra $a=1/2;b=-1/2$

Do đó $f(x)=\dfrac{x^2}{2}-\dfrac{x}{2}+c$

$f(n)=1+2+3+...+n$

Áp dụng điều ta vừa chứng minh được thì:
$f(1)-f(0)=1$

$f(2)-f(1)=2$

....

$f(n)-f(n-1)=n$

Do đó

$1+2+...+n=f(1)-f(0)+f(2)-f(1)+...+f(n)-f(n-1)=f(n)-f(0)=\dfrac{n^2}{2}-\dfrac{n}{2}=\dfrac{n(n-1)}{2}$




#495167 Chứng minh rằng $2(x^2+y^2+z^2)+xyz \ge 7 $

Đã gửi bởi Oral1020 on 25-04-2014 - 22:25 trong Bất đẳng thức và cực trị

Cho x;y;z không âm,thoảvx+y+z=3.Chứng minh rằng :$2(x^2+y^2+z^2)+xyz \ge 7 $




#495157 CMR: $\sum \frac{a^{2}b(b-c)}{a+b...

Đã gửi bởi Oral1020 on 25-04-2014 - 21:45 trong Bất đẳng thức - Cực trị

Cộng lần lượt từng biểu thức cho abc,ta được bđt tương đương với:
$\sum \dfrac{ab^2(a+c)}{a+b} \ge 3abc$

Bất đẳng thức trên luôn đúng theo AM-GM cho 3 số :D




#495043 $\boxed{\text{Chuyên Đề}}$ Bất đẳng thức - Cực trị

Đã gửi bởi Oral1020 on 25-04-2014 - 12:36 trong Bất đẳng thức và cực trị


156, 

c, Cho $x\epsilon \left [ 0;1 \right ].Tìm GTLN của P=x(1-x)^{3}$

 

 

$156.c$

Mình áp dụng dụng bất đẳng thức Cauchy cho :
$x(1-x)^3=\dfrac{3x(1-x)(1-x)(1-x)}{3} \le \dfrac{27}{256}$
dấu = xảy ra khi $x=\dfrac{1}{4}$

 

Chú ý: Trích đề




#495042 $\frac{a^{3}}{b}+\frac{b^...

Đã gửi bởi Oral1020 on 25-04-2014 - 12:26 trong Bất đẳng thức và cực trị

Bằng Cauchy-Schwart,ta đánh giá được:
$\sum \dfrac{a^3}{b} \ge \sum a^2$

Mặt khác ta lại có:
$a^2+b^2+c^2 \ge 2a+2b+2c-3$

$2(a^2+b^2+c^2) \ge 2(ab+ac+bc)$

Cộng vế theo vế và sử dụng giả thiết ta sẽ tìm được min của biểu thức




#494943 [Violympic9] Các bài toán violympic lớp 9 cho kì thi quốc gia sắp tới.

Đã gửi bởi Oral1020 on 24-04-2014 - 19:54 trong Cuộc thi VIOlympic (Cuộc thi do Bộ giáo dục và đào tạo tổ chức)

 

43) Cho hình thang cân ngoại tiếp hình tròn có độ dài 2 đáy là $\frac{2}{\pi}cm$ và $6cm$. Diện tích hình tròn nội tiếp hình thang bằng ...

 

44) Tam giác $ABC$ có $\widehat{B}=60^o$; $BC=8cm$; $AB+AC=12cm$. Tính $AB$
 

45) Tìm $x$ biết $x^2+\frac{4x^2}{(x+2)^2}=5$

 

46) Tính $x^4+y^4$ biết $\left\{\begin{matrix}x+y=4  &  & \\ (x^2+y^2)(x^3+y^3)=280  &  &  \end{matrix}\right.$

 

 

Tính đại nhá :) .
Do tứ giác ngoại tiếp nên AB+DC=AD+BC$=\dfrac{2}{\pi}+6$
Suy ra AD=$\dfrac{1}{\pi}+3$
Kẻ hai đường cao AH.Ta tính được $DH=\dfrac{6-\dfrac{2}{\pi}}{2}$
Từ đó ta tính được AH.Mà AH chính là đường kính của đường tròn đó => dt=3

 

Gọi tâm hình tròn nt là O, hình thang là ABCD

Kẻ OM, ON, OP lần lượt vuông góc vs AB, AD, DC

Chứng minh đc AOD là tam giác vuông

$\Rightarrow OM^{2}=MA.MD=\frac{3}{\pi }$

Diện tích hình tròn là 3 :)
@Oral:Mình bị lộn (đã sửa )




#494595 Cho $x>0 $ và $y\geq 0$ thoả mãn:$x^{3...

Đã gửi bởi Oral1020 on 22-04-2014 - 21:10 trong Bất đẳng thức và cực trị

Mình nghĩ là tìm GTLN chớ nhỉ (như vậy bạn nhé) :)




#494521 Cho x, y là hai số không âm thỏa mãn $x^{3}+y^{3}...

Đã gửi bởi Oral1020 on 22-04-2014 - 16:15 trong Bất đẳng thức và cực trị

a) Ta có : $x^3+y^3 \ge 0$

$\Rightarrow x-y \ge 0$ hay $x \ge y$

Mặt khác $x^3+y^3 \le x-y $

$\Leftrightarrow x-x^3 \ge y^3+y \ge 0$

$\Rightarrow x \ge x^3$

$\Rightarrow 1 \ge x^2$

Mà $x \ge 0$

$\Rightarrow x \le 1$

Từ đó ta suy ra đpcm

b)Do $x;y \in [0;1]$

$\Rightarrow x^3+y^3 \le x^2+y^2$

Ta có:

$ x- y \ge x^3+y^3 \ge x^3-y^3$

$\Leftrightarrow 1 \ge x^2+y^2+xy$

$\Leftrightarrow 1 \ge x^2+y^2$




#493219 $\frac{b^2+c^2-a^2}{bc}+\frac{c^2+a^2...

Đã gửi bởi Oral1020 on 15-04-2014 - 22:47 trong Bất đẳng thức và cực trị

Bất đẳng thức này tương đương với :

$(b+c-a)(a+b-c)(a+c-b) >0$
Luôn đúng với a;b;c là các cạnh của một tam giác :D




#493217 Đề thi học sinh giỏi cấp thành phố - Cần Thơ 2013-2014

Đã gửi bởi Oral1020 on 15-04-2014 - 22:38 trong Tài liệu - Đề thi

Do mình vẽ hình trên máy nên các điểm không giống như trong đề nha  (Có hai điểm H ) :D

a)Trước hết ta có:
$\widehat {KBC}+\widehat{KCB}=\widehat{KBD}+\widehat{KCE}+\widehat{DEC}+\widehat{HCB}$
Dễ thấy:

$\widehat{KBD}+\widehat{KCE}=\dfrac{\widehat{ABD}}{2}+\dfrac{\widehat{ACE}}{2}=\widehat{ABD}$

$\widehat{HBC}=\widehat{HAD}$

$\widehat{HCB}=\widehat{EAH}$

$\Rightarrow \widehat{KBD}+\widehat{KCE}=\widehat{ABD}+\widehat{BAD}=90$

$\Rightarrow$ K;D;C;B;E cùng nằm trên một đường tròn tâm I
Suy ra $\widehat{KIE}=\widehat{EBK}=\widehat{KCD}=\widehat{KID}$

Do đó hai cung EK và cung KD bằng nhau $\Rightarrow KE=KD$
b)Ta thấy H;K;I thẳng hàng do cùng nằm trên đường trung trực của ED.

Hình gửi kèm

  • Untitled.png



#471839 $\sqrt[8]{1-x}$ + $\sqrt[8]{x}...

Đã gửi bởi Oral1020 on 19-12-2013 - 23:13 trong Phương trình - hệ phương trình - bất phương trình

Ta có bất đẳng thức $\sqrt{a}+\sqrt{b} \ge \sqrt{a+b}$.Sử dụng liên tiếp là ra thôi :)

 

Giải phương trình:

 $\sqrt[8]{1-x}$ + $\sqrt[8]{x}$ =1




#471441 CM $F,E,P$ thẳng hàng

Đã gửi bởi Oral1020 on 17-12-2013 - 17:59 trong Hình học

Mình làm câu d) nhá :)
Gọi I là giao điểm của EF và AM $\Longrightarrow I$ là trung điểm của AM và $E;I;F$ thẳng hàng

Do đó,ta sẽ đy chứng minh $I;F;P$ thẳng hàng.

Ta có $IF //AC$ và $IP //AC$ do IP là đường trung bình của tam giác MAC$




#471440 Cho tứ giác $ABCD$ nội tiếp $(O;R)$ và ngoại tiếp $(...

Đã gửi bởi Oral1020 on 17-12-2013 - 17:49 trong Hình học

1524702_436715776428912_384587113_n.jpg

$\oplus$ Ta có các công thức sau đây:
$r^2 = \dfrac{(p-a)(p-b)(p-c)}{p}$

$AC'=AB'=p-a$

$BC'=BA'=p-b$

$CB'=CA'=p-c$

$\oplus$.Ta có: $IA^2.IB^2.IC^2=[(r^2+(p-a)^2][r^2+(p-b)^2][r^2+(p-c)^2]=\prod [\dfrac{(p-a)(p-b)(p-c)}{p}+(p-a)^2] = \dfrac{\prod bc(p-a)}{p^3}=\dfrac{a^2b^2c^2(p-a)(p-c)(p-b)}{p^3} \le \dfrac{a^2b^2c^2}{27}$

Do $(p-a)(p-b)(p-c) \le \dfrac{(3p-a-b-c)^3}{27}=\dfrac{p^3}{27}$ (AM-GM)

$\Longrightarrow IA.IB.IC \le \dfrac{abc}{3\sqrt{3}}$

 

 




#471424 Chứng min M là điểm tiếp xúc giữa đường tròn bàng tiếp ứng với góc A với BC

Đã gửi bởi Oral1020 on 17-12-2013 - 16:42 trong Hình học

Cho tam giác ABC ngoại tiếp đường tròn tâm I.Gọi E là điểm tiếp xúc của đường tròn với BC,kẻ đường kính EF.Đường thẳng AF cắt BC tại M.Chứng minh rằng M là điểm tiếp xúc giữa đường tròn bàng tiếp ứng với góc A với BC




#468998 $\sum \frac{ab}{a+3b+2c}\leq \fr...

Đã gửi bởi Oral1020 on 05-12-2013 - 12:56 trong Bất đẳng thức và cực trị

Áp dụng bất đẳng thức C-S,ta được:

$\sum \dfrac{ab}{a+3b+2c} = \sum \dfrac{ab}{(a+c)+(b+c)+2b} \le \sum \dfrac{ab}{9(a+c)}+\sum \dfrac{ab}{9(b+c)}+\sum \dfrac{a}{18} = \dfrac{a+b+c}{6}$




#466721 Chứng minh rằng AA1,BB1,CC1 đồng quy

Đã gửi bởi Oral1020 on 25-11-2013 - 18:51 trong Hình học

Nó là định lí Cevathì phải ?




#459884 $\frac{x^{4}}{y^{2}(z+x)}+...

Đã gửi bởi Oral1020 on 25-10-2013 - 16:19 trong Hình học

Áp dụng bất đẳng thức AM-GM,ta có:

$\dfrac{x^4}{x^2(y+z)}+\dfrac{x}{2}+\dfrac{x}{2}+\dfrac{y+z}{4} \ge 2x$

$\Longrightarrow \dfrac{x^4}{x^2(y+z)} \ge 2x-x-\dfrac{y+z}{4}$

Lấy $\sum$ hai vế,ta có đpcm




#456419 Chứng minh rằng $a+b+c+d$ chia hết cho 5

Đã gửi bởi Oral1020 on 09-10-2013 - 20:17 trong Số học

Cho a, b, c, d là các số nguyên thỏa mãn $a^{5}+b^{5}=4\left ( c^{5}+d^{5} \right )$

Chứng minh rằng  $a+b+c+d$ chia hết cho 5

Ta có:

$a^5+b^5=4(c^5+d^5)$

$\Longleftrightarrow a^5+b^5+c^5+d^5=5(c^5+d^5)$

$\Longrightarrow a^5+b^5+c^5+d^5 \vdots 5$

Mà $n^5-n \vdots 5$ (bạn tự chứng minh nhé :) )
$\Longrightarrow a^5+b^5+c^5+d^5 - (a+b+c+d)=5(c^5+d^5)-(a+b+c+d)$

$\Longleftrightarrow (a^5-a)+(b^5-b)+(c^5-c)+(d^5-d)=5(c^5+d^5)-(a+b+c+d)$

$\Longrightarrow a+b+c+d \vdots 5$